25

ALL INDIA PRELIMS TEST SERIES - 2020 · music of this form is a simple melodic extension of the raga in which it is composed. The Varnam is a beautiful creation of musical craftsmanship

  • Upload
    others

  • View
    2

  • Download
    0

Embed Size (px)

Citation preview

Page 1: ALL INDIA PRELIMS TEST SERIES - 2020 · music of this form is a simple melodic extension of the raga in which it is composed. The Varnam is a beautiful creation of musical craftsmanship
Page 2: ALL INDIA PRELIMS TEST SERIES - 2020 · music of this form is a simple melodic extension of the raga in which it is composed. The Varnam is a beautiful creation of musical craftsmanship

1 AIPTS 2020 (HIS - 08) (E) Answer Key Byju’s Classes: 9873643487

ALL INDIA PRELIMS TEST SERIES - 2020 MEDIEVAL INDIA + ART AND CULTURE OF MEDIEVAL INDIA +

CURRENT AFFAIRS OF JUNE 2019

ANSWER KEY 1. Ans. : (a)

Explanation: Dhrupad is a genre of Hindustani

classical music.

Statement 1 is correct: Dhrupad had an impetus

for popularity by the 14th century but it finds a

blossoming period from 15th century onwards to

about the 18th century. During these centuries we

meet the most respected and renowned singers

and patrons of this form. There was Man Singh

Tomar, the Maharaja of Gwalior. It was he who

was mainly responsible for the enormous vogue of

dhrupad.

Statement 2 is correct: Tansen was an

accomplished dhrupad singer. Swami Haridasa a

hermit of Brindavan was not only a dhrupadiya,

but one of the most central figures in the Bhakti

cult in the Northern areas of India. By tradition he

was the guru of Tansen, one of the best known

dhrupad singers and one of the nine jewels of

Emperor Akbar's court.

Statement 3 is incorrect: There were four

schools or vanis of singing the dhrupad. The

Gauhar vani developed the raga or unadorned

melodic figures. The Dagarvani emphasized

melodic curves and graces. The Khandar vani

specialised in quick ornamentation of the notes.

Nauhar vani was known for its broad musical

leaps and jumps. These vanis 'are now

indistinguishable. Compared to the vanis of the

dhrupads, we have gharanas, in the khyal. These

are schools of singing founded or developed by

various individuals or patrons such as kings or

noblemen. The oldest of these is the Gwalior

gharana.

2. Ans. : (a)

Explanation : Recently two Cabinet Committee

were constituted to meet the emerging

requirements of the nation.

Statement 1 is incorrect: Cabinet Committee on

Investment and Growth will identify key projects

required to be implemented on a time-bound

basis, involving investments of Rs. 1,000 crore or

more, or any other critical projects with regard to

infrastructure and manufacturing. Reviewing

economic trends and framing an integrated

economic policy is under purview of the Cabinet

Committee on Economic Affairs.

Statement 2 is correct: Cabinet Committee on

Employment and Skill Development will provide

direction to all policies, programmes for skill

development to increase the employability of the

workforce for effectively meeting the emerging

requirements of the rapidly growing economy and

mapping the benefits of demographic dividend.

3. Ans. : (b)

Explanation:

The course of the evolution of Indian music

saw the emergence of two different subsystems

as Hindustani and Carnatic music. The two

distinct styles, Hindustani and Carnatic came

into vogue after the advent of the Muslims,

particularly during the reign of the Mughal

Emperors of Delhi. Both the systems of music

received their nourishment from the same

original source. Whereas the Indian music of

the Northern part of India assimilated some

features of the music of the Persian and Arabic

musicians who adorned the courts of the

Mughal rulers of Delhi, the music of the South

continued to develop along its own original

lines. But the fundamental aspects of both the

systems of the North and South have been the

same.

It is said, that South Indian Music, as

known today, flourished in Deogiri the capital

city of the Yadavas in the middle ages, and

that after the invasion and plunder of the city

by the Muslims, the entire cultural life of the

city took shelter in the Carnatic Empire of

Vijayanagar under the reign of

Krishnadevaraya. Thereafter, the music of

South India came to be known as Carnatic

Music.

Statement 1 is incorrect: Gitam is the simplest

type of composition. Taught to beginners of

music, the gitam is very simple in construction,

with an easy and melodious flow of music. The

music of this form is a simple melodic extension of

the raga in which it is composed. The Varnam is a

beautiful creation of musical craftsmanship of a

high order, combining in itself all the

characteristic features of the raga in which it is

composed.

Page 3: ALL INDIA PRELIMS TEST SERIES - 2020 · music of this form is a simple melodic extension of the raga in which it is composed. The Varnam is a beautiful creation of musical craftsmanship

2 AIPTS 2020 (HIS - 08) (E) Answer Key Byju’s Classes: 9873643487

Statement 2 is correct: Jatisvaram-has no

sahitya or words. The piece is sung with solfa

syllables only. Examples of solfa syllables a taka

tari kita naka tatin gina tam

Statement 3 is correct: The Kirtanam had its

birth about the latter half of the 14th century. It is

valued for the devotional content of the sahitya.

Clothed in simple music, the kirtanam abounds in

Bhakti bhava. It is suited for congregational

singing as well as individual presentation.

4. Ans. : (d)

Explanation:

Many features of local cultures and traditions

were combined with Islamic architectural

practices during the medieval period. Thus, in

the field of architecture, a mix of many

structural techniques, stylised shapes, and

surface decorations came about through

constant interventions of acceptance, rejection

or modification of architectural elements.

These architectural entities or categories

showcasing multiple styles are known as Indo-

Saracenic or Indo-Islamic architecture.

Decorative forms in Indo-Saracenic

architecture included designing on plaster

through incision or stucco. They were

influenced by Turkish, Persian and Indian

traditions.

Statement 1 is correct : Tiles were also used to

surface the walls and the domes. Popular

colours were blue, turquoise, green and yellow.

Example : The tomb of Shah Rukn-i-Alam (Sufi

saint) was built by the Tughluq ruler of Delhi,

Ghiyas-ud-din (r. 1320-1325), between 1320 and

1324 AD. The exterior is the tomb is ornamented

with the use of carved brick and wood as well as

blue and white faience mosaic tiles with raised

relief patterns.

Statement 2 is correct : Profuse use of jalis is a

prominent feature of Indo-Saracenic architecture.

Jalis were also used in Hindu temples. A fine

example of Jali is seen at Tomb of Salim Chishti,

Fatehpur Sikri shows Islamic geometric patterns

developed in western asia. Jali is seen in

Pattadakal Virupaksha temple and Chola

architecture.

Statement 3 is correct : Carvings of bell and

chain motifs showcase Indian influence in

provincial styles of Indo - Islamic architecture.

Amongst provincial styles, the architecture of

Bengal and Jaunpur is regarded as distinct.

Gujarat was said to have a markedly regional

character for patrons borrowed elements from

regional temple traditions such as toranas, lintels

in mihrabs, carvings of bell and chain motifs, and

carved panels depicting trees, for tombs, mosques

and dargahs. The fifteenth century white marble

dargah of Shaikh Ahmad Khattu of Sarkhej in

Gujarat is a good example of provincial style and it

heavily influenced the form and decoration of

Mughal tombs.

Statement 4 is correct: Calligraphy was

extensively used for decoration. Islamic verses

were carved on the surfaces of Islamic

monuments. Example : Taj Mahal

5. Ans. : (d)

Explanation: Sufism is a common term used for

Islamic mysticism. The Sufis were very liberal in

their religious outlook. They believed in the

essential unity of all religions. They preached

spirituality through music and doctrines that

professed union with God. Sufism originated in

Iran and found a congenial atmosphere in India

under the Turkish rule. Their sense of piety,

tolerance, sympathy, concept of equality and

friendly attitude attracted many Hindus, mostly

from lower classes, to Islam. Sufi saints such as

Moinuddin Chisti, Nizamuddin Auliya, Fariduddin

Ganj-e-Shakar were the pioneer sufïs who are still

loved, respected and honoured in India

Pair 1 is correctly matched: Khanqah, the

institutions (abode of Sufis) set up by the Sufis in

northern India took Islam deeper into the

countryside.

Pair 2 is incorrectly matched: Takia (resting

place/retreat of a Muslim saint) also became the

centres for the propagation of Islamic ideas. These

were patronized both by the aristocracy and the

common people. The Sufis emphasized respect for

all human beings. Whirling meditation is a form of

physically active meditation which originated

among Sufis.

Pair 3 is correctly matched: Dargahs are shrines

built over the graves of revered Muslim saints

Pair 4 is correctly matched: The Sufis were

organised into religious orders or silsilahs. These

silsilahs were named after their founders such as

Chishti, Suhrawardi, Qadi. and Naqshbandis.

According to Abul Fazl, the author of the Ain-i-

Akbari, there were as many as fourteen silsilahs in

India during the sixteenth century. Each order had

its own khanqah, which served as a shelter for the

Sufi saints and for destitutes, and later developed

as a centre of learning.

6. Ans. : (a)

Explanation : Recently in India, South Asia‘s first

proton therapy centre, Apollo Proton Cancer

Centre (APCC) was inaugurated. It is located in

Chennai and offers world-class proton therapy to

cancer patients using pencil-beam scanning

technology with the highest degree of precision. It

is.

Page 4: ALL INDIA PRELIMS TEST SERIES - 2020 · music of this form is a simple melodic extension of the raga in which it is composed. The Varnam is a beautiful creation of musical craftsmanship

3 AIPTS 2020 (HIS - 08) (E) Answer Key Byju’s Classes: 9873643487

Statement 1 is correct: The difference between

X-rays and protons is that proton therapy

deposits much less radiation in the normal tissues

that are not being targeted. Radiation energy

causes changes in the cells that injure or destroy

them. Thus proton therapy minimizes damage to

cells.

Statement 2 is correct : Under this method,

proton beams are regulated and channelled

through a vacuum passageway to the precise site

of tumour, thereby resulting in minimal damage to

the surrounding healthy tissues and reduced side

effects.

Statement 3 is incorrect: Proton therapy is

effective against many kinds of cancers,

particularly effective on tumours affecting eye and

brain, spinal cord or other vital organs, head and

neck cancers, deep seated abdominal and pelvic

cancers, recurrent cancers and paediatric cancers.

7. Ans. : (b)

Explanation: Though illiterate Akbar patronised

scholars and learned men. In his court there were

nine such Navratna Mulla Do Pyaza, Hakin

Human, Abdur Rahim Khan e Khanan, Abul Tayal,

Tansen, Raja Todar Mal, Raja Man Singh, Faizi

and Birbal.

Statement 1 is correct: Abdur Rahim Khan e

Khanan was the son of Bairam Khan, Akbar's

trusted guardian and mentor, who was of Turkic

ancestry.

Statement 2 is correct: Raja Todar Mal was

instrumental in introducing a new system of

revenue known as Zabt and a system of taxation

called Dahsala. He was one of the 'Navratnas' of

Akbar's court.

Statement 3 is correct: Faizi was the brother of

Abul Fazl. Akbar highly recognised the genius in

him and appointed him tutor of his sons.Faizi is

remembered for his works of poetry, numbering in

the hundreds. One of his most well known books

is Tabashir al-Subh, a collection of poems.

Statement 4 is incorrect: Abul Fazl Mamuri was

a historian of the Mughal Empire during

Aurangzeb's reign and author of Tarikh-i-

Aurangzeb, Tarikh-i-Abul Fazl Mamuri and co-

author of Shahjahannama. It was Abu al-Fazal ibn

Mubarak also known as Abu'l-Fazl, Abu'l Fadl and

Abu'l-Fadl 'Allami who was the Grand vizier of the

Mughal emperor Akbar, and author of the

Akbarnama, the official history of Akbar's reign in

three volumes, and a Persian translation of the

Bible. He was also one of the Nine Jewels of

Akbar's royal court

8. Ans. : (c)

Explanation : The Bhakti saints attacked the

rigidity in religion and the objects of worship. They

disregarded caste and encouraged women to join

in their religious gatherings. The Bhakti saints did

their entire teaching in the local vernacular

language to make it comprehensible even to simple

minds.

Statement 1 is correct: Ramanuja was from the

South and he taught in the language of the

common people. Ramananda was his disciple.

Ramananda was born at Allahabad and educated

at Varanasi.

Statement 2 is incorrect: Ramananda took his

Guru‘s message to the northern parts of India.

Ramananda is thought to be the founder of the

Ramanandi Sampradaya. The members of this

community are known as Ramanandis, Vairagis or

Bairagis. They are known for their self-imposed

highly disciplined, austere, structured and simple

lifestyle.

Statement 3 is correct: Ramananda‘s followers

belonged to different walks of life. For example,

Kabir was a weaver, Sadhana was a butcher,

Ravidasa was a cobbler and Sena was a barber.

9. Ans. : (a)

Explanation : Sher Shah organized a brilliant

administrative system. The central government

consisted of several departments. The king was

assisted by four important ministers:

1. Diwan–i-Wizarat – also called as Wazir - in

charge of Revenue and Finance.

2. Diwan-i-Ariz – in charge of the Army.

3. Diwan-i-Rasalat- Foreign Minister.

4. Diwan-i-Insha- Minister for Communications.

Statement 1 is correct: Foreign Minister was

called Diwan-i-Risalat during Sher Shah‘s

administration.

Statement 2 is correct: The Revenue and Finance

minister was called Diwan -i- Wizarat.

Statement 3 is incorrect: The minister in

charge of the Army was called Diwan-i-Ariz.

The Minister for Communications was called

Diwan-i-Insha.

Statement 4 is incorrect: The Minister for

Communications was called Diwan-i-Insha. The

minister in charge of the Army was called Diwan-i-

Ariz.

10. Ans. : (d)

Explanation:

Statement 1 is correct: Ashtadiggajas is the

collective title given to the eight Telugu poets in

the court of the emperor Sri Krishna Deva Raya.

These poets were Allasani Peddana, Nandi

Thimmana, Madayyagari Mallana, Dhurjati,

Ayyalaraju Ramambhadrudu, Pingali Surana,

Ramaraja bhushanudu and Tenali

Ramakrishnudu.

Page 5: ALL INDIA PRELIMS TEST SERIES - 2020 · music of this form is a simple melodic extension of the raga in which it is composed. The Varnam is a beautiful creation of musical craftsmanship

4 AIPTS 2020 (HIS - 08) (E) Answer Key Byju’s Classes: 9873643487

Statement 2 is incorrect: The Rule of

primogeniture was not established in the kingdom

of Vijayanagar. Due to this there was confusion

regarding the succession to the throne. There was

a series of civil war among the various contenders

to the throne.

Statement 3 is correct: During the reign of

Rama Raya, the Battle of Talaikotta was fought

between the Vijayanagar Kingdom and combined

forces of Bijapur, Ahmadnagar, Golkonda and

Bidar. In this war, the Vijayanagar Kingdom was

defeated. This battle was generally considered to

mark the end of the Vijayanagar Empire. This

battle is also known as Raksasa Thangadi. Rama

Raya was imprisoned and executed. The city of

Vijayanagar was destroyed.

11. Ans. : (a)

Explanation :

Statement 1 is incorrect: The founder of the

Bahmani kingdom was Alauddin Bahman Shah

also known as Hasan Gangu. He founded the

kingdom in1347. Firoz Shah was not the founder

of this kingdom.

Statement 2 is incorrect: Mahamud Gawan

carried out many reforms in the Bahamani

kingdom. He divided the kingdom into eight

provinces called Tarafs. Each tarafs was governed

by a tarafdar. In every province, a tract of land

was set apart for the expenses of the sultan. Such

land was called khalisa.

Statement 3 is correct: Ahmad Wali Shah

shifted the capital from Gulbarga to Bidar.

Earlier the capital of Bahamani Kingdom was

Gulbarga.

12. Ans. : (d)

Explanation:

● The Australian government launched a

campaign ‗Zero Chance‘ to raise awareness

among people trying to enter the country

illegally by boats.

● The message of Zero Chance is simple. Anyone

who tries to come illegally to Australia by boat

has zero chance of success.

● It is also important in Indian context, as

few Indian citizens try to illegally enter

Australia but were detained and deported.

For example: there were instances where some

157 asylum-seekers from India were detained

at sea for weeks, after rejecting a return to

India.

● The group, thought to be mostly ethnic

Tamils from Sri Lanka which includes 50

children, who left from the Indian port of

Pondicherry but were intercepted by

Australian authorities.

13. Ans. : (c)

Explanation:

Statement 1 is correct : Construction of tall

Raya Gopurams and Kalyanamandapam with

carved pillars are the important features of this

School of Architecture. The sculptures on the

pillars were carved with distinctive features.

The horse was the most common animal found in

these pillars. Large mandapams contain one

hundred pillars as well as one thousand pillars in

some big temples. These mandapams were used

for seating the deity on festival occasions. Also,

many Amman shrines were added to the already

existing temples during this period.

Statement 2 is correct: Vittalaswamy and

Hazara Ramaswamy temples were the best

examples of this style of Architecture. The

Varadharaja and Ekambaranatha temples at

Kanchipuram stand as examples for the

magnificence of the Vijayanagara style of temple

architecture. The Raya Gopurams at

Thiruvannamalai and Chidambaram speak of the

glorious epoch of Vijayanagar.

14. Ans. : (d)

Explanation :

Option (a) is incorrect: The founder of the

Bahmani kingdom was Alauddin Bahman Shah

also known as Hasan Gangu. He was not granted

the title of Malik-ul-Tujjar.

Option (b) is incorrect: Malik Amber was a

popular Prime Minister of the Ahmadnagar

Sultanate. He is credited with carrying out a

revenue settlement of much of the Deccan, which

formed the basis for subsequent settlements. He

was not related to the Bahmani kingdom.

Option (c) is incorrect: Ahmad Wali Shah shifted

the capital from Gulbarga to Bidar. He was not

granted the title of Malik-ul-Tujjar

Option (d) is correct: The Bahmani kingdom

reached its peak under the guidance of Mahmud

Gawan.

● The success of Muhammad Shah was due to

the advice and services of his minister

Mahmud Gawan. He was also a military

genius.

● He waged successful wars against Vijayanagar,

Orissa and the sea pirates on the Arabian sea.

His conquests include Konkan, Goa and

Krishna-Godavari delta. Thus he expanded the

Bahmani Empire through his conquests.

● His administrative reforms were also

important. They were aimed to increase the

control of the Sultan over the nobles and

provinces.

● He was granted the title of Malik-ul-Tujjar.

Page 6: ALL INDIA PRELIMS TEST SERIES - 2020 · music of this form is a simple melodic extension of the raga in which it is composed. The Varnam is a beautiful creation of musical craftsmanship

5 AIPTS 2020 (HIS - 08) (E) Answer Key Byju’s Classes: 9873643487

● He built a magnificent madrasa or college in

the Capital Bidar. Some of the famous

scholars of the time belonging to Iran and Iraq

came to madrasa at the instance of Mahmud

Gawan.

15. Ans. : (d)

Explanation: The word Kathak has been derived

from the word Katha which means a story.

Kathakars or story-tellers, are people who narrate

stories largely based on episodes from the epics,

myths and legends. It probably started as an oral

tradition. Mime and gestures were perhaps added

later on to make the recitation more effective. Thus

evolved a simple form of expressional dance,

providing the origins of what later developed into

Kathak as we see it today.

Statement 1 is correct: With the coming of the

Mughals, this dance form received a new impetus.

A transition from the temple courtyard to the

palace durbar took place which necessitated

changes in presentation. In both Hindu and

Muslim courts, Kathak became highly stylised and

came to be regarded as a sophisticated form of

entertainment. Today, Kathak has emerged as a

distinct dance form. Being the only classical

dance of India having links with Muslim

culture, it represents a unique synthesis of

Hindu and Muslim genius in art. Further,

Kathak is the only form of classical dance wedded

to Hindustani or the North Indian music. Both of

them have had a parallel growth, each feeding and

sustaining the other.

Statement 2 is correct: The Kathak dancer is

accompanied by an equally dextrous percussionist

- the tabla player - whose fingers have to match

the speed of the dancer's feet and the sound of the

ghungroos. The dancer or the tabla player recite a

series of bols which are given physical

interpretation by dance and musical

accompaniment.

Statement 3 is correct: The nineteenth century

saw the golden age of Kathak under the patronage

of Wajid Ali Shah, the last Nawab of Oudh. He

established the Lucknow gharana with its strong

accent on bhava, the expression of moods and

emotions.Another noteworthy patron of Kathak

was Raja Chakradhar of Raigadh in Madhya

Pradesh. He encouraged the gharanas - Lucknow

as well as Jaipur.

16. Ans. : (c)

Explanation :

Pair 1 is correctly matched: Garba, also spelled

garaba, singular garbo, type of dance commonly

performed at festivals and on other special

occasions in the state of Gujarat, India. Garba

dances celebrate fertility, honour womanhood, and

pay respect to any of an array of mother

goddesses. Although men may participate on some

occasions, women are the typical performers of

garba. Dancing begins slowly and gradually

increases in speed.

Pair 2 is incorrectly matched: Ghoomra is a folk

song and dance genre of Orissa, prevalent in the

districts of Sambalpur, Kalahandi, Bolangir and

Cuttack. This dance is named after the musical

instrument ―Ghoomra (drum)‖. Dancers dance

while playing the ghoomra drum with both hands.

Pair 3 is correctly matched: Karagam is a folk

dance from Tamil Nadu. The dancers carry colorful

decorative pots, piled one after another on their

heads, while balancing the pots dancer gaily

executes intricate movements with their feet.

Goddess Mariamma (Goddess of health and rain),

the pots signifies the symbols for fertility.

Pair 4 is correctly matched: It is a dance form

of Mizoram. Bamboos are used in the performance

of the Cheraw dance. In the performance, the

dancer moves by stepping alternatively in and out

from between and across a pair of horizontal

bamboos.

17. Ans. : (d)

Explanation: The inscriptions of the Cholas who

ruled in Tamil Nadu refer to more than 400 terms

for different kinds of taxes.

Option (a) is incorrect: Land revenue was

referred to as kadamai.

Option (b) and (c) are incorrect: There were also

taxes on thatching the house, the use of a ladder

to climb palm trees, a cess on succession to family

property, etc, but they had no specific terms

mentioned in the inscription of Cholas.

Option (d) is correct: The most frequently

mentioned tax in the inscriptions of the Cholas is

vetti, which was not taken in cash, but in the form

of forced labour.

18. Ans. : (a)

Explanation: The middle ages witnessed a series

of battles which not only decided the fate of the

kingdoms but in some cases changed the very

course of history. These battles were not bound by

any spatial or temporal limitations but were rather

spread to the whole of the country. Able

generalship and military advances in tactics made

the warfare more than sheer numbers. While some

of them led to the seeding of a new dynasty or clan

in the society, some of them led to a virtual

wipeout from the map of the country.

Statement 1 is correct: Battle of Tarain was

fought between Prithviraj III of Chauhan Dynasty

and Muhammad of Ghazni. In the second battle

fought in 1192, Prithviraj lost the battle and the

chauhan power in Delhi came to an end. They

were then limited to a small principality near

Ajmer (Ajayameru).

Page 7: ALL INDIA PRELIMS TEST SERIES - 2020 · music of this form is a simple melodic extension of the raga in which it is composed. The Varnam is a beautiful creation of musical craftsmanship

6 AIPTS 2020 (HIS - 08) (E) Answer Key Byju’s Classes: 9873643487

Statement 2 is incorrect:

Battle of Chandawar was fought between

Jayachandra and Muhammad Ghori in the

year 1194. Defeat in this crucial battle paved

the way for Muhammad to increase his

conquest of the Ganga Valley and a gradual

spread of Turkish rule in the country.

The battle between Babur and Medini Rai was

fought at Chanderi in 1528 and helped babur

consolidate his position in north India.

Statement 3 is correct: The Battle of Kanauj was

fought between Sher Shah and Humayun in the

year 1540. It was here that Humayun was met

with a decisive defeat which left him as a king

without a kingdom and he had to seek shelter

abroad. Though accompanied by his younger

brothers, able generalship helped Sher Shah carry

the day and establish the reign of Sur dynasty over

Agra.

Statement 4 is incorrect: The battle of

Bannihatti or Talikota was fought in 1565 and

marked an end to the once almighty Vijayanagar

Empire. In the battle, a unified army of five Deccan

Sultanates(Ali adil shah and four others) defeated

the Vijayanagara Army and Vijayanagara General

Ramaraju was killed in the battlefield.

19. Ans. : (b)

Explanation: Basava (1105-1167) challenged

traditional Hindu beliefs and practices relating to

caste, ritual pollution, and status of women. The

movement started by him called Virashaivism

projected for its followers a new social order which

was based on ritual equality (in terms of worship

and belief), sanctity of all work, and universal

ritual purity i.e. purity of all followers irrespective

of sex, age and occupation. This movement utilised

Kannada, the spoken language of the masses, to

communicate its ideas.

Statement 1 is incorrect: Basava founded the

Virashaiva or Lingayat sect. Simultaneous with the

development of Virashaivism was the growth and

expansion of another movement of similar beliefs.

This was the Aradhya Shaiva movement in Andhra

(Cuddapah and Kurnool district mainly) and

Kannada region (Mysore). It was started by

Mallikarjuna Pandita Aradhya, who was a

contemporary of Basava in the twelfth century.

Statement 2 is correct: Basava was a minister in

the court of the Kalachuri king, Bijjala of Kalyana

in north Karnataka in the twelfth century (AD

1160). Bijjala and Basava were said to have had

numerous differences. He left the royal court and

founded a movement for reforming society.

Statement 3 is correct: From the middle of the

12th century, Virasaivism began to influence the

people and literature of the Kannada speaking

region. The religious literary work of Basava is

known as Vachanas. The works of Basava and his

followers constitute an important contribution to

the medieval Kannada literature.

20. Ans. : (b)

Explanation:

―Progress of the World‘s Women 2019-2020:

Families in a Changing World‖ by UN Women

examines how the transformations in families

impact women‘s rights.

According to the report, women continue to

enter the labour market in large numbers, but

marriage and motherhood reduce their labour

force participation rates, and the income and

benefits that come with it.

The report calls on policymakers and people in

all walks of life to transform families into

places of equality and justice—where women

can exercise choice and voice, and where they

have physical safety and economic security.

21. Ans. : (a)

Explanation: Chola inscriptions mention several

categories of land as follows-

Pair 1 is incorrectly matched: Vellanvagai was

land of non-Brahmana peasant proprietors, and

land gifted to temples was referred to as devadana

or tirunamattukkani.

Pair 2 is correctly matched: Pallichchhandam

was land donated to Jaina institutions.

Pair 3 is incorrectly matched: Tirunamattukkani

was and gifted to temples, and land gifted to

Brahmanas was referred to as brahmadeya.

Additional Information : Shalabhoga was

referred to land for the maintenance of a school.

22. Ans. : (b)

Explanation: Many foreign travelers visited the

Vijayanagar Empire and their accounts are also

valuable. The Moroccan traveler, Ibn Battuta,

Venetian traveler Nicolo de Conti, Persian

traveler Abdur Razzak and the Portuguese

traveler Domingo Paes were among them who left

valuable accounts on the socio-economic

conditions of the Vijayanagar Empire.

Ibn Battuta

● Visited India during the period of 1333-1347.

● His book, called Rihala, provides valuable

information about the socio-economic

condition of Indian Subcontinent in the

fourteen century.

● He left an account of Harihara I’s reign in his

book Rehla.

Nicolo de Conti

● Visited India during the period of 1420-1421

A.D.

Page 8: ALL INDIA PRELIMS TEST SERIES - 2020 · music of this form is a simple melodic extension of the raga in which it is composed. The Varnam is a beautiful creation of musical craftsmanship

7 AIPTS 2020 (HIS - 08) (E) Answer Key Byju’s Classes: 9873643487

● He visited Vijayanagar Kingdom during the

time of Dev Raya II. He left an account in the

Travels of Nicolo Conti.

● Given a graphic account of Vijayanagar

capital.

Abdur Razzak

● Visited the VijayaNagar kingdom during the

reign of Dev Raya II.

● He gives an account of the reign of Devaraya

II in his Matla as Sadain Wa Majma ul

Bahrain.

Marco Polo

● He had not visited the Vijayanagar Empire. He

visited South India in 1294 A.D during the

reign of Pandyan ruler of Madurai,

Madverman, Kulshekhara The Vijaynagar

empire was established in 1336.

23. Ans. : (b)

Explanation :

Sufism:

● Sufism was a liberal reform movement

within Islam. While orthodox Muslims

emphasise external conduct, the Sufis lay

stress on inner purity. While the orthodox

believe in blind observance of rituals, the Sufis

consider love and devotion as the only means

of attaining salvation.

● Institutionally, the sufis began to organise

communities around the hospice or

khanqah (Persian) controlled by a teaching

master known as shaikh (in Arabic), pir

or murshid (in Persian). He enrolled

disciples (murids) and appointed a successor

(khalifa).

Statement 1 is incorrect: Be-shari‘a sufi did not

comply with Sharia. Ba-shari‗a sufis complied

with sufism. Some mystics initiated the

movements based on the radical interpretation of

Sufism. They observed extreme forms of ascetism

like mendicancy, scorning the khanqah. Such

mystics often referred to Be-shari'a as they did not

comply with Shari‘a. They are also known by

different names such as Qalandars, Madaris,

Malangs and Haidaris.

Statement 2 is correct: Sufism highlighted the

importance of pir or guru for spiritual

development. According to Sufism, one must have

the guidance of a pir or guru, without which

spiritual development is impossible.

Statement 3 is correct : Ideas emphasised by

Sufism are meditation, good actions, repentance

for sins, performance of prayers and pilgrimages,

fasting, charity and suppression of passions by

ascetic practices.

24. Ans. : (b)

Explanation :

● Almost all types of puppets are found in India.

Puppetry throughout the ages has held an

important place in traditional entertainment.

Puppets from different parts of the country

have their own identity. Regional styles of

painting and sculpture are reflected in them.

● Stories adapted from puranic literature, local

myths and legends usually form the content of

traditional puppet theatre in India which, in

turn, imbibes elements of all creative

expressions like painting, sculpture, music,

dance, drama, etc. The presentation of puppet

programmes involves the creative efforts of

many people working together.

● There are different types of puppets

o String Puppets

o Shadow Puppets

o Rod Puppets

o Glove Puppets

Shadow Puppetry

● India has the richest variety of types and

styles of shadow puppets. Shadow puppets are

flat figures. They are cut out of leather, which

has been treated to make it translucent.

● Shadow puppets are pressed against the

screen with a strong source of light behind it.

The manipulation between the light and the

screen makes silhouettes or colourful

shadows, as the case may be, for the viewers

who sit in front of the screen.

● This tradition of shadow puppets survives in

Orissa. Kerala, Andhra Pradesh, Karnataka,

Maharashtra and Tamil Nadu.

Statement 1 is incorrect

● The traditional puppets of Rajasthan are

known as Kathputli. It is an example of

string puppetry

● The Kathputli is accompanied by a highly

dramatised version of the regional music.

● Puppeteers manipulate them with two to five

strings which are normally tied to their fingers

and not to a prop or a support.

Statement 2 is incorrect

● In Kerala, the traditional glove puppet play is

called Pavakoothu.

● It came into existence during the 18th century

due to the influence of Kathakali, the famous

classical dance-drama of Kerala, on puppet

performances.

● The manipulator puts his hand into the bag

and moves the hands and head of the puppet.

The theme for Glove puppet plays in Kerala is

based on the episodes from either the

Ramayana or the Mahabharata.

Page 9: ALL INDIA PRELIMS TEST SERIES - 2020 · music of this form is a simple melodic extension of the raga in which it is composed. The Varnam is a beautiful creation of musical craftsmanship

8 AIPTS 2020 (HIS - 08) (E) Answer Key Byju’s Classes: 9873643487

Statement 3 is correct

● Ravanachhaya is a theatrically exciting puppet

theatre of Orissa. It is an example of shadow

puppetry.

● The puppets are in one piece and have no

joints. They are not coloured, hence throw

opaque shadows on the screen.

● The manipulation requires great dexterity,

since there are no joints.

● The puppets are made of deer skin and are

conceived in bold dramatic poses.

Statement 4 is correct:

● Tholu Bommalata, Andhra Pradesh's shadow

theatre has the richest and strongest

tradition.

● The puppets are large in size and have jointed

waist, shoulders, elbows and knees.

● They are coloured on both sides. Hence, these

puppets throw coloured shadows on the

screen.

● The music is dominantly influenced by the

classical music of the region and the theme of

the puppet plays are drawn from the

Ramayana, Mahabharata and Puranas.

25. Ans. : (a)

Explanation:

Statement 1 is correct: Chauth and

sardeshmukhi were the taxes collected not in the

Maratha kingdom but in the neighbouring

territories of the Mughal empire or Deccan

sultanates.

Statement 2 is incorrect: Chauth was one fourth

of the land revenue paid to the Marathas in order

to avoid the Maratha raids. Sardeshmukhi was an

additional levy of ten percent on those lands which

the Marathas claimed hereditary rights.

Statement 3 is incorrect: Sardeshmukhi was an

additional levy of ten percent on those lands which

the Marathas claimed hereditary rights. Chauth

were such a type of land revenue which were paid

to Marathas in order to avoid Maratha raids.

26. Ans. : (b)

Explanation :

Bhakti Movement:

● Medieval Bhakti movement was the direct

result of the influence of the spread of Islam in

India.

● Preaching of Sufi teachers shaped the thinking

of Bhakti reformers like Ramananda, Kabir

and Nanak.

● Main features of the Bhakti movement were

the Condemnation of rituals and ceremonies,

rejection of caste system and idol worship, and

intense love and devotion.

● Impact of Bhakti movement was - awakening

among Hindus regarding the futility of

Superstitions and promotion of religious

tolerance.

● Promotion of regional languages as Bhakti

Saints preached in the regional language.

Statement 1 is incorrect: According to

Sankaracharya, all the objects in creation are

illusory in nature. He propounded the idea of

Advaita. According to this concept, God is without

attributes. As the Advaita concept is related to

Nirgunabrahman (God without attributes).

Statement 2 is correct: Ramanuja preached the

idea of Visishtadvaita. According to him God is

Sagunabrahman. God, soul, matter are real. But

God is inner substance and the rest are his

attributes. He also advocated prabattimarga or

path of self-surrender to God. He invited the

downtrodden to Vaishnavism.

Statement 3 is correct: Madhava from Kannada

region propagated Dvaita or dualism of Jivatma

and Paramatma. According to his philosophy, the

world is not an illusion but a reality. God, soul,

matter are unique in nature.

27. Ans. : (b)

Explanation: Any means of payment that exists

purely in electronic form is called Digital money.

Digital money is exchanged using technologies

such as smartphones, credit cards and online

cryptocurrency exchanges. It is not tangible like a

dollar bill or a coin.

● Facebook along with its partners is leaping

into the world of cryptocurrency with its own

digital money called Libra.

● Cryptocurrency is based on block chain

technology. The most important feature of a

cryptocurrency is that it is not controlled by

any central authority: the decentralized nature

of blockchain makes cryptocurrency

theoretically immune to the old ways of

government control and interference.

● Facebook, with its over 2 billion users, and its

partners pose a challenge since it can shift

power from central banks towards

multinational corporations.

● The new digital currency wallet launched by

Facebook is called Calibra.

28. Ans. : (a)

Explanation : Building monumental forts with

embattlements was a regular feature in medieval

times, often symbolising the seat of power of a

king. Some examples of strong, complex edifices

which still exercise the imagination of the visitor

are the forts of Chittor, Gwalior, Daulatabad,

earlier known as Devgiri and Golconda. There were

several strategic devices used in the forts to

confound the enemy

Page 10: ALL INDIA PRELIMS TEST SERIES - 2020 · music of this form is a simple melodic extension of the raga in which it is composed. The Varnam is a beautiful creation of musical craftsmanship

9 AIPTS 2020 (HIS - 08) (E) Answer Key Byju’s Classes: 9873643487

Statement 1 is correct: Daulatabad had several

strategic devices to confound the enemy, such as

staggered entrances so that gates could not be

opened even with the help of elephants. It also had

twin forts, one within the other but at a higher

elevation and accessed by a complex defence

design arrangement. One wrong turn in the

labyrinth or complex pathway could lead to the

enemy soldier going in circles or falling to his

death several hundred feet below.

Statement 2 is correct: Concentric circles of

outer walls were employed in Golconda, so that the

enemy had to breach these at all stages before

getting in.

Statement 3 is incorrect:

Chittorgarh bears the distinction of being the

largest fort in Asia and was occupied for the

longest length of time as the seat of power. But

the fort was deserted in light of the

introduction of artillery in the 16th century,

and therefore the capital was shifted to more

secure Udaipur, located on the eastern flank of

Aravalli hill range, where heavy artillery &

cavalry were not effective.

On the other hand, the Jaigarh fort situated in

the Aravalli hill range features a cannon

named "Jaivana" (Jaivana Cannon), which was

manufactured in the fort precincts. The fort

was built by Jai Singh II in 1726 to protect the

Amer Fort and its palace complex.

29. Ans. : (d)

Explanation: The pala empire was founded by

gopala in around 750 AD and was the main power

centre in eastern India for a couple of centuries.

The period saw the growth of art and culture in the

region as well as trade relations with the

neighbouring areas.

Statement 1 is incorrect : The relations with

southeast asian nations were that of a close one.

Trade prospered which brought about some

prosperity in the Pala territory as well. The

relations were not only economical but cultural as

well. The powerful Sailendra dynasty in South

East Asia sought permission to build a monastery

at Nalanda which was subsequently granted.

Statement 2 is correct : Dharmapala was one of

the main rulers in the pala dynasty who was

devoted to the upkeep of learning institutes. He

not only revived the Nalanda university but also

set apart 200 villages for its upkeep. He went on to

establish the Vikramshila University which

became second only to Nalanda in fame.

Statement 3 is correct : Gopala was elected as a

king by the notable men of the area in order to end

the state of anarchy which was prevailing in the

region around 750 AD thus leading to the founding

of the Pala dynasty.

Statement 4 is incorrect : The title of Adivaraha

was adopted by the Pratihara rulers as they were

devotees of Vishnu.

30. Ans. : (a)

Explanation : Muhammad bin Tughlaq was in

some ways one of the most remarkable rulers of

his age. He was not liked by many orthodox

theologians who accused him of being a

‗rationalist‘. Unfortunately he was inclined to be a

bit hasty and impatient as a result of which many

of his experiments oftentimes met with devastating

failures.

Statement 1 is correct : One of the steps taken

by Muhammad Tughlaq was the introduction of

the ‗token currency‘. A shortage of silver in the

fourteenth century and a successful

implementation of token currency by Qublai Khan

of China encouraged Tughlaq to go ahead with the

move. Bronze coins were introduced which were

supposed to have the same value as the silver

tanka. However this method could not succeed due

to the lack of quality standards.

Statement 2 is incorrect : The Qarachil

expedition of Tughlaq was launched in the

Kumaon hills of the Himalayas. After a few

successes, the soldiers ventured too far into the

mountains and suffered a disaster. It is said that

from an army of 10,000, only 10 persons returned.

Statement 3 is incorrect : Diwan-i-amir-i-kohi

was set up as a separate department by

Muhammad Bin Tughlaq for the purpose of

extending and improving agriculture in the doab

region. Loans were provided to the cultivators to

induce them to cultivate superior crops. There was

a large scale misappropriation of money due to

which the experiment failed.

31. Ans. : (a)

Explanation: Bronze sculptures and statues of

Buddhist, Hindu and Jain icons have been

discovered from many regions of India dating from

the second century until the sixteenth century.

Most of these were used for ritual worship and are

characterised by exquisite beauty and aesthetic

appeal.

Statement 1 is correct: Shiva is associated with

the end of the cosmic world with which this

dancing position is associated. In the Chola period

Nataraja bronze sculpture Shiva has been shown

balancing himself on his right leg and suppressing

the apasmara, the demon of ignorance or

forgetfulness, with the foot of the same leg. At the

same time he raises his left leg in bhujangatrasita

stance, which represents tirobhava, that is kicking

Page 11: ALL INDIA PRELIMS TEST SERIES - 2020 · music of this form is a simple melodic extension of the raga in which it is composed. The Varnam is a beautiful creation of musical craftsmanship

10 AIPTS 2020 (HIS - 08) (E) Answer Key Byju’s Classes: 9873643487

away the veil of maya or illusion from the devotee‘s

mind. His four arms are outstretched and the

main right hand is posed in abhaya hasta or the

gesture suggesting. The upper right holds the

damaru his favourite musical instrument to keep

on the beat tala. The upper left hand carries a

flame while the main left hand is held in dola

hasta and connects with the abhaya hasta of the

right hand. His hair locks fly on both the sides

touching the circular jvala mala or the garland of

flames which surrounds the entire dancing

figuration.

Statement 2 is incorrect: In Buddhist centres

like Nalanda, a school of bronze casting emerged

around the ninth century during the rule of the

Pala Dynasty in Bihar and Bengal regions. In the

gap of a few centuries the sculptors at Kurkihar

near Nalanda were able to revive the classical style

of the Gupta period. A remarkable bronze is of a

four-armed Avalokitesvara, which is a good

example of a male figure in graceful tribhanga

posture. The tribhanga is a standing body position

which means three parts break, consisting of three

bends in the body - at the neck, waist and knee

and hence the body curves into a gentle "S" shape.

It is considered the most graceful and sensual.

Avalokiteshwara is the Bodhisattva of

compassion : Avalokiteshvara supremely

exemplifies the bodhisattva‘s resolve to postpone

his own buddhahood until he has helped every

sentient being on earth achieve liberation

(moksha; literally, ―release‖) from suffering

(dukkha) and the process of death and rebirth

(samsara).

32. Ans. : (a)

Explanation:

Statement 1 is correct:

● The National People‘s Party (NPP) is the the

first from the north-eastern region to become a

national party.

● The NPP is recognised as a State party in

Arunachal Pradesh, Manipur, Meghalaya and

Nagaland.

● It became the 8th national political party in

the country.

● The other national political parties are: Indian

National Congress, Bharatiya Janata Party,

Bahujan Samaj Party, Communist Party of

India, CPI (Marxist), National Congress Party

and All India Trinamool Congress.

Statement 2 is incorrect:

● A party is recognized as a national party by

the Election Commission of India.

● In its order, the ECI said that the NPP was

given the national party status for fulfilling

conditions such as polling more than 6% of

the total votes in the last general election and

being recognised as a State party in at least

four States.

33. Ans. : (d)

Explanation: Krishnadeva Raya belonged to the

Tuluva dynasty. Krishnadeva Raya‘s rule was

characterised by expansion and consolidation.

This was the time when the land between the

Tungabhadra and Krishna rivers (the Raichur

doab) was acquired (1512), the rulers of Orissa

were subdued (1514) and severe defeats were

inflicted on the Sultan of Bijapur (1520).

Krishnadeva Raya is credited with building some

fine temples and adding impressive gopurams to

many important south Indian temples. He also

founded a suburban township near Vijayanagar

called Nagalapuram after his mother.

Additional Information:

Harihar I: Harihar along with Bukka I was the

founder of the Vijayanagar kingdom. He ruled

from 1336 AD to 1356 AD.

Bukka I: He ruled from 1356 AD- 1377 AD

for about 20 years after Harihar I. He is

known for expanding the empire by defeating

the Shambuvaraya kingdom of Arcot and

the Reddis of Kondaveedu. He defeated

extended the territory into the south till

Rameswaram.

Dev Raya II: He ruled from 1406-1422 AD.

He is known for battling against the

Bahmani Sultan of Gulbarga, Reddis of

Kondaveedu and Velamas of Telangana,

and also for retaining the territories he

controlled.

34. Ans. : (c)

Explanation: Dashavatar is the most developed

theatre form of the Konkan and Goa regions. The

performers personify the ten incarnations of Lord

Vishnu-the god of preservation and creativity. The

ten incarnations are Matsya (fish), Kurma

(tortoise), Varaha (boar), Narsimha (lion-man),

Vaman (dwarf), Parashuram, Rama, Krishna (or

Balram), Buddha and Kalki. Apart from stylized

make-up, the Dashavatar performers wear masks

of wood and papier mache.

Statement 1 is correct: Dashavatar is the most

developed theatre form of the Konkan and Goa

regions.

Statement 2 is incorrect:

● In most theatres as in the case of Dashavatar,

the roles of the female characters are mostly

played by males.

● Tamaasha is an exception. It is a traditional

folk theatre form of Maharashtra. Unlike other

theatre forms, in Tamaasha the female actress

is the chief exponent of dance movements in

the play. She is known as Murki.

Page 12: ALL INDIA PRELIMS TEST SERIES - 2020 · music of this form is a simple melodic extension of the raga in which it is composed. The Varnam is a beautiful creation of musical craftsmanship

11 AIPTS 2020 (HIS - 08) (E) Answer Key Byju’s Classes: 9873643487

Statement 3 is correct: Apart from stylized

make-up, the Dashavatar performers wear masks

of wood and papier mache.

35. Ans. : (a)

Explanation:

Statement 1 is correct: Manor was the castle

where the Lord lived. The land of the lord was to

be tilled by serfs along with their own lands. This

system did not exist in India.

Statement 2 is incorrect: The serf could not

migrate or marry without the permission of the

Lord. He could also not change his profession.

Such a system was not generally prevalent in

India.

Additional Information : Since these are

considered important features of feudalism, some

scholars argue that feudalism did not exist in

India.

36. Ans. : (b)

Explanation: Inscriptions from Uttaramerur in

Chingleput district, Tamil Nadu, provide details of

the way in which the sabha ( assemblies of

agraharas) was organised. The sabha had separate

committees to look after irrigation works, gardens,

temples, etc.

Statement 1 is incorrect: Names of those eligible

to be members of these committees were written

on small tickets of palm leaf and kept in an

earthenware pot, from which a young boy was

asked to pick the tickets, one by one for each

committee. They were not directly elected.

Statement 2 is correct: All those who wish to

become members of the sabha/assembly of the

agraharas were to be owners of land from which

land revenue was collected. The committee

members were members of the assembly.

Statement 3 is correct: If anyone had been a

member of any committee in the last three years,

he could not become a member of another

committee.

37. Ans. : (a)

Explanation:

Statement 1 is correct: Recently, Saudi Arabia

has become the first Arab country to be granted

full membership of the Financial Action Task Force

(FATF).

Statement 2 is correct: It is an inter-

governmental body established by the Ministers of

its Member jurisdiction in the G-7 Summit that

was held in Paris in 1989. The objectives of the

FATF are to set standards and promote effective

implementation of legal and operational measures

for combating money laundering, terrorist

financing and other related threats to the integrity

of the international financial system.

Statement 3 is incorrect: FATF has put Pakistan

on its terrorist financing watch list or the Grey List

for failing to curb terror financing. Israel is one of

the 39 members of FATF. Grey list is a warning

given to the country that it might come in Black

list. If a country is unable to curb terror financing

and money laundering it is shifted from grey list to

black list.

38. Ans. : (d)

Explanation:

Statement 1 is correct : Slave market existed for

both men and women. During the Delhi Sultanate

period, large sections of town consisted of slaves

and domestic servants. Slaves were generally

bought for domestic services.

Statement 2 is correct : They could also rise to

high offices and even be promoted to king position.

For example Some of rulers of the slave dynasty,

for example Qutbud-din-Aibak, were slaves in the

earlier period. Skilled slaves were valued and some

of them rose to even rose to high offices.

Statement 3 is correct : Slaves were allowed to

marry, and to own some personal property.

Condition of the slave was better than that of a

domestic servant because the master of the former

was obliged to provide him food and shelter, while

a free person may starve to death. However, it was

widely accepted that slavery was degrading.

Additional Information : Qutubuddin Aibak was

a slave who rose to the rank of Sultan.

39. Ans. : (a)

Explanation :

Statement 1 is correct : Sher Shah‘s ray was

scientific to an extent and not based on guess

work. The different rates were calculated by

computing the average produce from the sown

land

Statement 2 is correct : Shershah introduced the

Patta and Kabuliyat (or Qabuliyat) system of land

deeds. Under the Patta system, the area sown,

types of crops cultivated and revenue share was

duly written on paper. The Qabuliyat system

involved a deed agreement between the peasant

and the state.

Statement 3 is incorrect : The dagh system was

introduced by Alauddin Khalji. This was a system

which was enforced in order to bring about quality

control as far as the recruitment of the cavalry was

concerned.

Statement 4 is incorrect : The dak chowki

service was prevalent during the period of the

Delhi Sultanate. This method was however widely

used by Sher Shah in order to keep himself

informed about the developments in the far off

areas.

Page 13: ALL INDIA PRELIMS TEST SERIES - 2020 · music of this form is a simple melodic extension of the raga in which it is composed. The Varnam is a beautiful creation of musical craftsmanship

12 AIPTS 2020 (HIS - 08) (E) Answer Key Byju’s Classes: 9873643487

Additional Information : The dagh system made

sure that horses of inferior quality may not be

substituted. The dak chowki service was used to

gather news/intelligence.

40. Ans. : (a)

Explanation: The Chhattisgarh government is

processing habitat rights for Abujh Marias, a

Particularly Vulnerable Tribal Group (PVTG) under

Forest Rights Act 2006. The PVTGs are a section of

scheduled tribes who are socio-economically and

educationally backward and are relatively isolated.

75 tribal groups have been categorized by the

Ministry of Home Affairs as Particularly Vulnerable

Tribal Groups (PVTG)s.

Statement 1 is correct: Abujh Marias belong to a

PVTG community.

Statement 2 is correct: Abujhmarh, where this

tribe lives, is considered by the government to be

one of the last remaining strongholds of Left-wing

extremism.

Statement 3 is incorrect: It was only in 2009

that the Government of Chhattisgarh lifted the

restriction on the entry of common people in the

area imposed in the early 1980s.

41. Ans. : (a)

Explanation:

Statement 1 is correct: Cholas led successful

naval expeditions to Sri Lanka, Maldives and Sri

Vijaya empire(South east Asia). The conquest of

Maldives was one of the naval exploits of Rajaraja

(985-1014 A.D). He also invaded Sri Lanka and

annexed its northern part to the empire. Naval

expeditions of the Cholas were meant to further

indian ocean trade which brought them high

revenues.

Statement 2 is correct: They sent diplomatic and

commercial embassies to China. These were partly

diplomatic and partly commercial. Chola

embassies reached China in 1016 and 1033. A

Chola embassy of 70 merchants reached China in

1077 and received 81,800 string of copper cash in

return for articles like glassware, ivory, camphor,

brocades etc.

Statement 3 is incorrect: A Shailendra king (of

Sri Vijaya empire) was allowed to build a Buddhist

monastery in Nagapattinam and a village was

endowed for its upkeep. This exhibited their

tolerant character which actually helped promote

Buddhism in the region.

42. Ans. : (d)

Explanation: A number of important changes took

place in Indian society during this period. One of

these was the growing power of a class of people

who arc variously called samanta, rartak, rautia

(rajput), etc., by the contemporary writers. Their

origins were very different as-

Statement 1 is correct: Some were government

officers who were increasingly paid not in cash but

by assigning to them revenue-bearing villages.

Statement 2 is correct: Some were defeated rajas

and their supporters, who continued to enjoy the

revenue of limited areas.

Statement 3 is correct: Others were tribal or clan

leaders. Also, some others were local hereditary

chiefs' or military adventurers who had carved out

a sphere of authority with the help of armed

supporters.

Additional information : The actual position of

these sections varied. Some of them were only

village chiefs, some of them dominated a tract

comprising a number of villages, while a few

dominated an entire region. They constantly

contended against each other, and tried to

enhance their sphere of authority and privileges.

43. Ans. : (c)

Explanation :

Statement 1 is correct: The Konkan was a

narrow strip of land between the Western Ghats

and the Arabian Sea. It was extremely fertile.

Statement 2 is incorrect: Within the port,

Konkan included Goa which was an important

outlet for the products of the region. Motupalli port

was an important port of the Kakatiyas, located on

the eastern coast, in Andhra Pradesh.

Statement 3 is correct: India was not breeding

good quality horses and was dependent on imports

from west Asia. The sea trade was therefore very

important for the southern states. Sea port was

also important for the import of horses from Iran

and Iraq.

Additional Information : Similarly the

Tungabhadra doab and the Marathwada country

were coveted by both the Bahamanis and the

Vijayanagara rulers.

44. Ans. : (c)

Explanation :

Statement 1 is correct: A number of travellers

mention the prevalence of Sati in different regions

of the country. Ibn Batuta, the famous Moroccan

traveller of the 14th century, records that he had

witnessed with horror an instance of the practice

of Sati. According to him, permission of the sultan

had to be taken for the performance of Sati.

Statement 2 is incorrect: The practice of Purdah

was adopted from Greece and Iran by the Turks

and brought to India. The growth of the system

has been attributed to the fear of Hindu women

Page 14: ALL INDIA PRELIMS TEST SERIES - 2020 · music of this form is a simple melodic extension of the raga in which it is composed. The Varnam is a beautiful creation of musical craftsmanship

13 AIPTS 2020 (HIS - 08) (E) Answer Key Byju’s Classes: 9873643487

being captured by the invaders. Perhaps the most

important factor was that the purdah had become

a symbol of the higher classes in society and it was

worn by upper caste women as a symbol of

prestige. The practice became widespread among

upper caste women. Purdah was not traditionally

observed by lower-class women.

Statement 3 is correct: Widow remarriage is

included among the practices prohibited in the

Kali Age. But this apparently applied to the three

upper castes only.

45. Ans. : (a)

Explanation :

Statement 1 is correct: Gopurams are lofty gates

that pierce the high wall that surrounds the

temple. As a temple got wealthier, they expanded

their immediate periphery as a result of which

there were new gopurams which were erected.

Hence, one could come up with a scenario where a

temple complex could have multiple gopurams.

Statement 2 is incorrect: Mandapas have flat

roofs and carved pillars. Dance performances of

devadasis used to be conducted here. They were

used for many other cultural themes and

programmes. They could be huge pavilions

constructed outdoors or could be housed in the

covered temple complex. The temple deity is

housed in the garbhagriha which is located in the

main structure itself.

Additional Information : Brihadeshwara temple

of Tanjore is an excellent example.

46. Ans. : (a)

Explanation:

Statement 1 is correct: Price regulation lowered

the prices of supplies needed by the army. This

allowed Alauddin to maintain a large army. A large

army was required to withstand the Mongol threat.

Secret agents called Munhias sent reports to the

sultan about the functioning of the markets.

Statement 2 is correct: He realised land revenue

in cash which allowed him to pay his soldiers in

cash. He was the first to pay his soldiers in cash.

Measurement of land ensured that nobody evades

land tax.

Statement 3 is incorrect: Cattle tax (Chari) and

house tax (Ghari) were also levied by Alauddin

Khalji. Audits were conducted of accounts of

revenue officials to prevent corruption. The

intermediaries were not allowed to exploit the

peasantry - their privileges were removed.

47. Ans. : (d)

Explanation :

Statement 1 is correct:

● The Sultan made government posts hereditary.

This made the Sultanat dependent on a

narrow oligarchy of a small number of

families.

● He also extended the principle of hereditary to

the army as well.

Statement 2 is correct:

● He repaired and dug a number of canals. The

longest canal was about 200 Kilometre which

took off from the river Sutlej to Hansi; another

canal took off from the Yamuna.

● These canals were meant for irrigation

purposes.

Statement 3 is correct: He set up a department

for public works and built many towns and canals.

Hisar was one such town.

Additional Information : Firuz‘ policy of

appeasement secured the Sultanat for the short

term. But did more harm than good in the long

term.

48. Ans. : (d)

Explanation : The Vijayanagar Empire was one of

the mightiest empires established south of the

Vindhyas. In terms of wealth and opulence the

kingdom knew no bounds. They were in a constant

tussle with the neighbouring kingdom of Bahmani

for control of the Tungabhadra doab and the

konkan. Most of the modern day Karnataka,

Andhra Pradesh, Tamil Nadu and Kerala were part

of the Vijayanagara empire.

49. Ans. : (c)

Explanation: Recently, the central Government

has given states and Union Territories June 30,

2020 deadline, to roll out the ‗one nation, one

ration card‘ system.

Statement 1 is correct:

Page 15: ALL INDIA PRELIMS TEST SERIES - 2020 · music of this form is a simple melodic extension of the raga in which it is composed. The Varnam is a beautiful creation of musical craftsmanship

14 AIPTS 2020 (HIS - 08) (E) Answer Key Byju’s Classes: 9873643487

● The beneficiaries can buy subsidized food

grains in any part of the country.

● Ration card Aadhar linkage is a must to access

the scheme.

Statement 2 is correct:

● A person will only be eligible for the subsidies

supported by the centre such as those under

Nation Food security act, 2013.

● Even if a beneficiary moves to a state where

grains are given for free he/she will not be able

to access those benefits.

Statement 3 is incorrect:

● Though subsidized food grains can be bought

using a single ration card, the beneficiaries

can buy subsidized food grains only in Fair

Price Shop having fully online electronic Point

of Sale devices.

● This is for biometric/Aadhaar authentication.

Statement 4 is correct : A migrant will be allowed

to buy a maximum of 50% of the family quota.

This is to ensure that the individual, after shifting

to another place does not buy the entire family

quota in one go.

50. Ans. : (c)

Explanation:

Statement 1 is correct: Paper manufacture was

introduced in India by the Turks. It had been

discovered by the Chinese in the 2nd century. It

was known in the Arab world in the 5th century,

and travelled to Europe only during the 14th

century.

Statement 2 is correct: The production of textiles

was improved by the introduction of spinning

wheels during this period. Cotton could be cleaned

faster and better by the use of cotton carder‘s bow.

This enabled a better and faster means of

production of the cotton textiles.

Contribution of Delhi Sultanate Period :

● Improvement of the Rahat so that water could

be lifted from a deeper level for irrigation.

● The other crafts included paper making, glass

making, the spinning wheel and an improved

loom for weaving.

● Introduction of Superior mortar which enabled

the Turks to erect the magnificent building.

Additional Information : Indian textiles were of

great demand in China and West Asia during this

period.

51. Ans. : (d)

Explanation:

Statement 1 is correct:

● The Nadu was a kinship based social

organisation marked by common agrarian

features. It consisted of subsistence level

settlements coming together for common

economic and social activities.

● The integration of the nadus into a larger and

systematic agrarian organisation was achieved

through land grants to brahmanas

(brahmadeya) and the temple by the ruling

families (Pallavas, Pandyas and Cholas).

● The Brahmana assembly called the Sabha

was responsible for the construction of

irrigation works, advancement of irrigation

technology and the management of the land

granted to Brahmanas.

Statement 2 is correct: The assembly of elders in

the settlements on the lands which were not

granted to Brahmanas were called Ur. It also had a

managerial function.

Statement 3 is incorrect: The Sabha functioned

through various committees called variyams.

Statement 4 is correct:

● Revenue surveys and assessment of land

revenue were systematically undertaken under

the Cholas, in the eleventh century. In the

process, new and larger revenue units were

formed by grouping some nadus together and

even by partitioning some under different

valanadus.

● This was determined by their irrigational

needs and hence valanadus had consciously

chosen boundaries such as water courses.

● The valanadu was an artificial unit and a

politico economic division created by the will of

a political authority. The valanadus were

named after the kings who created them.

Note : The terms like nadu and ur denoted the

agrarian settlements themselves and the

assemblies of people who managed them.

52. Ans. : (b)

Explanation: Jan Shikshan Sansthan aims to

provide vocational training to non-literates, neo-

literates as well as school drop-outs in rural

regions by identifying skills that have a relevant

market in that region.

Statement 1 is correct: It aims to transform

labour based economy to a knowledge based

economy.

Statement 2 is incorrect: Over two-thirds of

India‘s population comprises rural citizens.

The objective of JSS is to uplift this rural

population economically by imparting essential

skills training.

Statement 3 is incorrect: It is an initiative of the

Ministry of Skill Development and

Entrepreneurship.

Page 16: ALL INDIA PRELIMS TEST SERIES - 2020 · music of this form is a simple melodic extension of the raga in which it is composed. The Varnam is a beautiful creation of musical craftsmanship

15 AIPTS 2020 (HIS - 08) (E) Answer Key Byju’s Classes: 9873643487

53. Ans. : (c)

Explanation :

● Kiosks, eaves and balconies built in the

Sultanate period were adopted from

Rajasthani and Gujarati architectural styles.

● Placing the tomb of a king in the midst of a

garden was a Lodi innovation in India. But the

charbagh ( 4 gardens) concept was used for

the first time in India by the Mughals

Additional Information : The charbagh concept

on a monumental scale can be found in the

Humayun‘s tomb in Delhi.

54. Ans. : (d)

Explanation:

Statement 1 is correct:

● A decade-long wait for getting geographical

indication (GI) has ended. The approval for GI

tag was jointly received by Karnataka and

Maharashtra.

● The controller general of patents, designs and

trademarks has granted the GI tag to 4

districts of Maharashtra (Kolhapur, Sangli,

Satara and Solapur) and 4 districts of

Karnataka (Dharwad, Belgaum, Bagalkot and

Bijapur).

Statement 2 is correct:

● These leather chappals are hand-crafted and

tanned using vegetable dyes. The art of

making them is passed down from one

generation to another.

● Once GI tag is granted no other producer can

misuse the name to market similar products.

55. Ans. : (c)

Explanation: RBI has relaxed the leverage ratio

(LR) for banks in a bid to help them expand their

lending activities. The leverage ratio is defined as

the Tier-I capital (core capital) as a percentage of

the bank‘s exposures.

Statement 1 is correct:

● The leverage ratio has been reduced to 4% for

domestic, systemically important banks, and

3.5% for other banks.

● Lowering leverage ratio will allow banks to

lend more from the same amount of capital

they hold, giving them an impetus to lend

more.

Statement 2 is correct: The Basel Committee on

Banking Supervision (BCBS) introduced the

concept of a leverage ratio in Basel III reforms to

indicate the level of debt incurred by banks. It was

introduced post 2008 global financial crisis to

increase the resilience of banks.

56. Ans. : (c)

Explanation: Shivaji was also a great

administrator. He laid the foundations of a sound

system of administration. The king was the pivot of

the government. He was assisted by a council of

ministers called Ashtapradhan. However, each

minister was directly responsible to Shivaji.

Following is the details of the different ministry

and its functions

1. Peshwa – Finance and general administration.

Later he became the prime minister.

2. Sar-i-Naubat or Senapati – Military

commander, a honorary post.

3. Amatya – Accountant General.

4. Waqenavis – Intelligence, posts and household

affairs.

5. Sachiv – Correspondence.

6. Sumanta – Master of ceremonies.

7. Nyayadhish – Justice.

8. Panditrao – Charities and religious

administration

Pair 1 is correctly matched: Amatya‘s function

was of Accountant General.

Pair 2 is incorrectly matched: Panditarao‘s work

was related to Charities and religious

administration. Sumanta had worked as a Master

of ceremonies.

Pair 3 is correctly matched: Sachiv‘s work was

related to correspondence.

Pair 4 is correctly matched: Waqenavis‗s work

was related to Intelligence, posts and household

affairs.

57. Ans. : (c)

Explanation: The territories of the empire were

divided into jagir, khalisa and inam. Income

from the Khalisa villages went directly to the royal

exchequer. Inam lands were those which were

allotted to learned and educated men. Jagirs were

allotted to nobles and members of the royal

families including queens.

58. Ans. : (c)

Explanation:

● Pietra dura or pietre dure, called parchin kari

or parchinkari in the Indian Subcontinent, is a

term for the inlay technique of using cut and

fitted, highly polished colored stones to create

images. It is considered a decorative art.

Page 17: ALL INDIA PRELIMS TEST SERIES - 2020 · music of this form is a simple melodic extension of the raga in which it is composed. The Varnam is a beautiful creation of musical craftsmanship

16 AIPTS 2020 (HIS - 08) (E) Answer Key Byju’s Classes: 9873643487

● A tessellation (mosaic designs) of a flat surface

is the tiling of a plane using one or more

geometric shapes, called tiles, with no overlaps

and no gaps. These are mostly restricted to

indo-islamic architectures.

59. Ans. : (a)

Explanation:

Option (a) is incorrect. Therefore, answer is (a)

Akbar reorganised the central machinery of

administration on the basis of division of power

between various departments, and of checks and

balances. While the post of wakil was not

abolished, it was stripped of all power and became

largely decorative. The post was given to important

nobles from time to time, but they played little part

in the administration. The head of the revenue

department continued to be the wazir. He was

generally not a person who held a high position

in the nobility.

Option (b) is correct : The diwan was

responsible for all income and expenditure, and

held control over khalisa, and inam lands.

Option (c) is correct : The head of the military

department was called mir bakhshi. It was the

mir bakshi and not the diwan who was

considered the head of the nobility.

Option (d) is correct : The chief officers of the

sarkar were the faujdar and the amalguzar, the

former being in charge of law and order, and the

latter responsible for the assessment and

collection of land revenue.

60. Ans. : (b)

Explanation: The Umbrella revolution of 2014 was

a protest for getting the universal suffrage that

Hong Kong was promised in the Basic Law, the

constitution that lays out how the city is to be

governed after its return from Britain to Chinese

sovereignty in 1997. Hong Kong recently faced a

redux of such an umbrella revolution.

● Hong Kong is now part of China under the

―one country, two system principle. Under

this, the city of Hong Kong remains a semi-

autonomous region.

● Lakhs of protesters marched the streets of

Hong Kong in the biggest protest since the

Umbrella Revolution to reject interference from

Beijing and demand for democracy.

● The protest erupted when a bill was proposed

by Hong Kong Chief Executive Carrie Lam to

extradite criminals to mainland China.

● There is a firm concern that law will target

political opponents in Hong Kong and there is

a fear that extradited suspects would likely

face torture in China‗s flawed justice system.

● It was called "the Umbrella Movement"

because protesters used umbrellas to protect

themselves from the tear gas used by police.

61. Ans. : (c)

Explanation : The areas sown, the type of crops

cultivated and the amount each peasant had to

pay was written down on a paper called ‗patta‘ and

each peasant was informed of it. During Sher

Shah‘s reign, no one was allowed to charge

anything extra.

Additional Information : The ‗patta‘ granted

security to the peasant from extortions from

zamindars.

62. Ans. : (c)

Explanation:

Statement 1 is incorrect: Babur introduced a

new mode of warfare in India. Although gunpowder

was known in India earlier, Babur showed what a

skilled combination of artillery and cavalry could

be achieved.

Statement 2 is incorrect: Sher Shah did not

make many changes in the administrative division

prevailing since the Sultanate period. He

apparently continued the central machinery of

administration which had been prevailing during

the Sultanate period. He did not favour leaving too

much authority in the hands of the ministers.

63. Ans. : (a)

Explanation:

Statement 1 is correct:

● The amended Foreigners (Tribunal) Order,

2019 also empowers individuals to approach

the Tribunals.

● ―Earlier only the State administration could

move the Tribunal against a suspect, but with

the final NRC about to be published and to

give adequate opportunity to those not

included, this has been done. If a person

doesn‘t find his or her name in the final list,

they could move the Tribunal,‖ said a senior

official.

Statement 2 is incorrect:

● The amendment has empowered district

magistrates in all States and Union Territories

to set up tribunals to decide whether a person

staying illegally in India is a foreigner or not.

In other words all states can now constitute

foreigners tribunals.

● The amended order also allows District

Magistrate to refer individuals who haven‘t

filed claims against their exclusion from NRC

to the Tribunals to decide if they are foreigners

or not.

Page 18: ALL INDIA PRELIMS TEST SERIES - 2020 · music of this form is a simple melodic extension of the raga in which it is composed. The Varnam is a beautiful creation of musical craftsmanship

17 AIPTS 2020 (HIS - 08) (E) Answer Key Byju’s Classes: 9873643487

64. Ans. : (b)

Explanation:

Statement 1 is correct: Humayun built a new

city near Delhi, named Dinpanah. The building

of Dinpanah was meant to impress friends and

foes alike. It could also serve as a second capital

in case, Agra was threatened by Bahadur Shah

who, in the meantime, had conquered Ajmer and

overrun eastern Rajasthan.

Statement 2 is incorrect: Dinpanah was built on

the banks of the river Yamuna.

Statement 3 is correct: Sher Shah, who

overthrew Humayun in 1540, razed Din Panah to

the ground and built his new capital, the Sher

Shahi, now known as Purana Qila fort, in

southeastern Delhi.

65. Ans. : (d)

Explanation:

Statement 1 is incorrect : It was in the zabti

system, and not batai system, where not only

were local prices taken into account, parganas

having the same type of productivity were grouped

into separate assessment circles. Thus, the

peasant was required to pay on the basis of

local produce as well as local prices. As soon as

the area sown by the peasant had been measured

by means of the bamboo linked with iron rings,

the peasant as well as the state knew what the

dues were. The peasant was given remission in the

land revenue if crops failed on account of drought,

floods, etc.

Statement 2 is incorrect: It was under the batai

system, and not the jabti system, where the

peasants were given the choice of paying in cash or

in kind, though the state preferred cash. In case of

crops such as cotton, indigo, oil-seeds, sugar-

cane, etc., the state demand was invariably in

cash. Hence, these were called cash crops.

66. Ans. : (c)

Explanation: Delhi became an important city only

in the twelfth century(medieval period).

Statement 1 is incorrect:

● Delhi first became the capital of a kingdom

under the Tomara Rajputs.

● Tomara Rajputs were defeated in the middle of

the twelfth century by the Chauhans (also

referred to as Chahamanas) of Ajmer.

● It was under the Tomaras and Chauhans that

Delhi became an important commercial centre.

Many rich Jaina merchants lived in the city

and constructed several temples. Coins minted

here, called dehliwal, had a wide circulation.

Statement 2 is correct:

● The transformation of Delhi into a capital that

controlled vast areas of the subcontinent

started with the foundation of the Delhi

Sultanate in the beginning of the thirteenth

century.

● The Delhi Sultans built many cities in the area

that we now know as Delhi.

● Dehli-i-Kuhna was the first city built by the

Delhi Sultans

● Quwwat al-Islam mosque and Qutab minar are

located here

Statement 3 is correct: Alauddin constructed a

new garrison town named Siri for his soldiers.

Statement 4 is incorrect: Muhammad bin

Thughlaq built a new city by enclosing the areas

lying between the cities of Siri, Tughlaqabad and

Lal Kot in 1334 A.D. The city was named

Jahanpanah, the asylum of the world. The city had

13 gates.

67. Ans. : (b)

Explanation:

● The National Auto Policy (draft) calls for

developing a roadmap for reduction in CO2

emissions through Corporate Average Fuel

Economy (CAFE) regulations.

● CAFE regulations aim to lower fuel

consumption (or improving fuel efficiency) of

vehicles by lowering carbon dioxide (CO2)

emissions.

● The norms are applicable for petrol, diesel,

LPG and CNG passenger vehicles. CAFE

regulations in India came into force from April

1, 2017.

68. Ans. : (b)

Explanation:

● The inscriptions issued between the eighth

and the thirteenth centuries in the Deccan

speak of the emergence of several major and

minor ruling powers such as the

Rashtrakutas, Chalukyas, Silaharas,

Kakatiyas, Sevunas (Yadavas), Hoysalas, etc.

● The main source of the income of these

states was agricultural taxation. Other

important sources of state income included

customs, excise duties and charges levied

on trade and industry. Regular offerings

and tributes by the feudatory chiefs

comprised another source of the income of the

state.

Statement 1 is incorrect: The villagers and

townsmen were also subject to some additional

imposts over the land tax and excise tax (the tax

on production).Upakriti and Kanika refers to a kind

of customary tax levied by the government on

villagers and townsmen in return for some service

performed for their benefit by the king or their

officers.

Page 19: ALL INDIA PRELIMS TEST SERIES - 2020 · music of this form is a simple melodic extension of the raga in which it is composed. The Varnam is a beautiful creation of musical craftsmanship

18 AIPTS 2020 (HIS - 08) (E) Answer Key Byju’s Classes: 9873643487

Statement 2 is correct: Sunkamu or Sunka was

a term that denoted duties on exports and imports

- excise duties and customs duties collected on

articles of merchandise brought to and taken from

market towns.

69. Ans. : (b)

Explanation: The Pala, Pratihara and Rashtrakuta

empires consisted of areas administered directly

and those ruled over by vassal chiefs. The directly

administered territories were divided into smaller

units.

Statement 1 is incorrect: The directly

administered territories in the Pala and Pratihara

empires were divided into:

● Bhukti (Provinces) under uparika

● Mandala/Visaya (Districts) under visayapati

● Pattala (unit below district)

Statement 2 is correct: The directly administered

territories in the Rashtrakuta empire were divided

into:

● Rashtra (province) under Rashtrapati

● Visaya (district)

● Bhukti (smaller unit of district)

70. Ans. : (b)

Explanation:

Statement 1 is incorrect : Bulk of the Mughal

nobles were drawn from the homeland of

mughals i.e Turan and from its neighbouring

areas, Tajikistan, Khorasan, Iran, etc. Mughal

rulers never followed a narrow racist policy.

Indian muslims also became the leading nobles

and were called Shaikhzadas or Hindustani.

From the time of Akbar ,Hindus also began to be

inducted into the nobility on a regular basis.The

largest section among them was that of the

Rajputs.

Statement 2 is incorrect : The Zamindar was

not the owner of all the lands comprising his

zamindari. The peasants who actually cultivated

the land could not be dispossessed as long as

they paid the land revenue. Thus the Zamindars

and the peasants had their own hereditary rights

in land.

Statement 3 is correct : In mughal

administration the petty officials were generally

paid in cash. Many others ,especially the

scholars, religious divine etc. were granted small

tracts of land for maintenance. Such grants were

called madaddimaash in mughal terminology. In

addition to the mughal emperor, local rulers and

zamindars, even nobles made such grants.

71. Ans. : (b)

Explanation: Amir Khusrau who was given the

title of nayak or master of both the theory and

practice of music, introduced many persian and

arabic airs (ragas), such as aiman, ghora,

sanam, etc. He is credited with having invented

the sitar, though we have no evidence of it.

72. Ans. : (a)

Explanation: Scientists from the University of

Kerala have decoded the genetic make-up of

Arogyapacha (Trichopus zeylanicus).

Statement 1 is incorrect : It is a highly potent

medicinal plant endemic to the Agasthya hills.

Statement 2 is correct.

● This ‗miracle plant‘ is known for its traditional

use by the Kani tribal community to combat

fatigue.

● Studies have also proved its varied spectrum

of pharmacological properties such as

antioxidant, aphrodisiac, anti-microbial, anti-

inflammatory, immunomodulatory, anti-

tumour, anti-ulcer, anti-hyperlipidemic,

hepatoprotective and anti-diabetic.

73. Ans. : (d)

Explanation:

Statement 1 is correct : Adil shah II was deeply

interested in music and composed a book called

kitab-i-nauras in which songs were set to various

musical modes or ragas. He built a new capital,

Nauraspur,in which a large number of musicians

were invited to settle. In his songs ,he freely

invoked the goddess of music and learning,

saraswati.

Statement 2 is correct :Due to broad approach

of Adil Shah II he came to be called ―jagat

Guru‖.He was solicitous of poor and had the title

of ―abla baba‖ or Friend of the poor.He accorded

patronage to all including Hindu saints and

temples. This include grants to pandharpur, the

centre of worship of Vithoba ,which became the

centre of the Bhakti movement in Maharashtra.

Statement 3 is correct : In the field of

architecture Muhammed Quli Qutb Shah

constructed many buildings ,the most famous of

which is the Char Minar.

74. Ans. : (c)

Explanation : Land which remained under

cultivation almost every year was called Polaj.

When it remained uncultivated, it was called

Parati(fallow). Land which had been fallow for

two to three years was called chachar, and for

longer than that, banjar. These were assessed at

concessional rates, the revenue demand gradually

rising till the full or polaj rate was paid in the fifth

or the eighth year. In this way, the state helped in

bringing virgin or uncultivated wasteland, under

cultivation.

Page 20: ALL INDIA PRELIMS TEST SERIES - 2020 · music of this form is a simple melodic extension of the raga in which it is composed. The Varnam is a beautiful creation of musical craftsmanship

19 AIPTS 2020 (HIS - 08) (E) Answer Key Byju’s Classes: 9873643487

75. Ans. : (a)

Explanation:

Pair 1 is correctly matched: Ibn Battuta‘s book

of travels, called Rihla, written in Arabic, provides

extremely rich and interesting details about the

social and cultural life in the subcontinent in the

fourteenth century. This Moroccan traveller was

born in Tangier into one of the most respectable

and educated families known for their expertise in

Islamic religious law or shari‗a. True to the

tradition of his family, Ibn Battuta received literary

and scholastic education when he was quite

young.

Pair 2 is correctly matched : Al-Biruni was born

in 973, in Khwarizm in present day Uzbekistan.

Khwarizm was an important centre of learning,

and Al-Biruni received the best education available

at the time. He was well versed in several

languages: Syriac, Arabic, Persian, Hebrew and

Sanskrit. Although he did not know Greek, he was

familiar with the works of Plato and other Greek

philosophers, having read them in Arabic

translations. In 1017, when Sultan Mahmud

invaded Khwarizm, he took several scholars and

poets back to his capital, Ghazni; Al-Biruni was

one of them. He arrived in Ghazni as a hostage,

but gradually developed a liking for the city, where

he spent the rest of his life until his death at the

age of 70. It was in Ghazni that Al-Biruni

developed an interest in India.

Pair 3 is incorrectly matched : François Bernier,

a Frenchman, was a doctor, political philosopher

and historian. Like many others, he came to the

Mughal Empire in search of opportunities. He was

in India for twelve years, from 1656 to 1668, and

was closely associated with the Mughal court, as a

physician to Prince Dara Shukoh, the eldest son of

Emperor Shah Jahan, and later as an intellectual

and scientist, with Danishmand Khan, an

Armenian noble at the Mughal court.

Pair 4 is correctly matched: One of the most

famous was the French jeweller Jean-Baptiste

Tavernier, who travelled to India at least six times.

He was particularly fascinated with the trading

conditions in India, and compared India to Iran

and the Ottoman empire.

76. Ans. : (b)

Explanation:

● Gujarat has launched India‗s first Emission

Trading Scheme to combat particulate air

pollution. The aim is to reduce particulate

pollutants PM2.5 and PM10.

● The programme is a market-based system

where the government sets a cap on emissions

and allows industries to buy and sell permits

to stay below the cap.

● The whole Emission Trading Scheme

programme has its basis in the polluter pays

principle as there is a cap on emissions and if

exceeds the polluter has to pay to buy permits.

77. Ans. : (a)

Explanation:

Statement 1 is correct : The shaivite nayanars

and and the Vaishnavite alvars disregarded the

austerities preached by the jains and the buddhist

and preached personal devotion to god as a means

of salvation. They disregarded the rigidities of the

caste system and carried their message of love and

personal devotion to god to various parts of south

india by using the local languages.

Statement 2 is incorrect : The poet saints of the

bhakti movement discarded the sanskrit language

and adopted the local language of the region to

preach their message of love and personal devotion

to god to various strata of the people.

78. Ans. : (c)

Explanation : The Bahamani ruler, Ahmad Shah I

shifted the capital from Gulbarga to Bidar. So the

sultanate that arose in that region was called

Bidar sultanate.

Additional Information

Bidar Sultanate was succeeded by Adil Shahi

dynasty

Golconda Sultanate of Qutb Shahi dynasty

was succeeded by the Mughal empire

Bijapur sultanate was ruled by Adil Shahi

dynasty

Ahmadnagar sultanate was succeeded by the

Mughal empire

Berar sultanate was annexed by the Adil

Shahis

79. Ans. : (a)

Explanation :

Statement 1 is correct : He abolished Jizya(tax

imposed on non-Muslims) and cow slaughter.

Hindus occupied high offices in his government.

He patronised Persian and Sanskrit. He was

looked upon with great reverence in his kingdom.

Kalhana‘s treatise Rajatarangini brings about the

Page 21: ALL INDIA PRELIMS TEST SERIES - 2020 · music of this form is a simple melodic extension of the raga in which it is composed. The Varnam is a beautiful creation of musical craftsmanship

20 AIPTS 2020 (HIS - 08) (E) Answer Key Byju’s Classes: 9873643487

greatness and tolerance of the king in an opulent

manner. He was known by his subjects as Bud

Shah or the great king.

Statement 2 is correct : Zaina Lanka is an

artificial island on Wular lake on which he built

his palace and a mosque. It was an architectural

marvel of the time as the logistical details which

had to be taken care of in order for the

construction to take place were immense.

Statement 3 is incorrect : He defeated the

Mongols when they invaded Ladakh. This helped

consolidate his power militarily as well.

80. Ans. : (a)

Explanation :

Statement 1 is correct : Arches and domes

became popular in India after the coming of the

Turks. Free standing arches and domes were first

used in Roman architecture. The Arabs borrowed

them from Rome. They were first introduced in

India by the Turks

Statement 2 is correct : Indian architecture

involved the construction through a lintel and

beam model whereby the bricks or the stones were

kept in such a way that one brick supported the

other through the base. This is the reason that

Indian architectural buildings did not include

domes as they could not withstand the pressure of

their own weight. They needed fine and strong

cement, otherwise the stones could not be held in

place. So the Turks used fine quality light mortar

in their buildings. This resulted in the sudden

growth of dome architecture in the country.

Statement 3 is incorrect : No human or animal

figures were used since it was considered to be un-

Islamic. However there was a prevalence of

geographical shapes and designs such as the

triangle and the semi circle.

81. Ans. : (c)

Explanation:

● The Regional Cooperation Agreement on

Combating Piracy and Armed Robbery against

Ships in Asia (ReCAAP) is the first regional

Government-to-Government agreement to deal

with piracy and armed robbery at sea in Asia.

● Presently 20 countries are members of

ReCAAP.

● India played an active role in the setting up

and functioning of ReCAAP Information

Sharing Centre (ISC) along with Japan and

Singapore.

82. Ans. : (c)

Explanation :

Statement 1 is correct : Guru nanak preached

the existence of one god and his teachings were

aimed at bridging distinctions between the Hindus

and the Muslims. He denounced idol-worship and

pilgrimages

Statement 2 is correct : Sikhism is a creed based

on the ideas of the Guru. He wanted to work as a

bridge between the Hindus and the Muslims and

thereby improve harmony and peace. He laid

emphasis on a middle path whereby a spiritual life

could be followed with the duties of the household.

Additional Information : The liberal teachings of

saints like Kabir and Guru Nanak created opinions

and ideas which continued to work through the

succeeding centuries.

83. Ans. : (d)

Explanation :

Statement 1 is correct : Sher Shah restored the

old imperial road called the Grand Trunk Road

from the river Indus in the west to Sonargaon in

Bengal. He also built a road from Agra to Jodhpur

and Chittor, linking up with the road to the

Gujarat sea ports. He built a third road from

Lahore to Multan. Multan was at that time the

staging point of caravans going to west Asia and

central Asia.

Statement 2 is correct : In his entire kingdom,

goods paid customs duty only at two places, goods

produced in Bengal or imported from outside paid

the customs duty at the border of Bengal and

Bihar at Sakrigali. Goods coming from central Asia

and west Asia paid customs duty at the Indus. No

one was allowed to levy taxes at roads, ferries or

towns anywhere else. Duty was paid second time

at the sale of the goods.

Statement 3 is correct : Currency reforms of

Sher Shah helped in the growth of commerce. He

struck fine coins of silver, gold and copper of

uniform standard in place of the earlier debased

coin of mixed metal. His silver rupee was so well

executed that it remained a standard coin for

centuries to come.

Additional Information : He also built sarais

(resting houses) along the roads at regular

intervals. Land in surrounding villages were

assigned for their upkeep. Village headmen and

zamindars were made responsible for any loss that

merchants suffered on the roads.

84. Ans. : (c)

Explanation: The Indian trading classes were well

organized and highly professional. Some

specialised in long distance, inter-regional trade

and some local, retail trade.

● The term Beoparis or Banik refers to retail

traders.

● In addition to retailing goods, the baniks had

their own agents, in the villages and

Page 22: ALL INDIA PRELIMS TEST SERIES - 2020 · music of this form is a simple melodic extension of the raga in which it is composed. The Varnam is a beautiful creation of musical craftsmanship

21 AIPTS 2020 (HIS - 08) (E) Answer Key Byju’s Classes: 9873643487

townships, with whose help they purchased

food-grains and cash crops.

● Seth, Bohra refers to those who were

specialised in long distance, inter-regional

trade.

85. Ans. : (a)

Explanation:

● Rajputs were a new section that arose in

society during the medieval period. During this

period, a large number of states were being

ruled by the Rajputs. These Rajputs were

leaders of clans which dominated certain

tracts of land, and provided the core of the

armed forces. The leaders of the clan, most of

whom were related to the ruler/king by ties of

blood, considered the state to be jointly ruled

by them.

● In parts of Rajasthan, tribal lands were

colonised and Brahmanas, traders and

warriors settled on the land. The tribal chiefs

continued to hold considerable power. This

was accompanied by the introduction of a

superior type of economy based on irrigation

and superior crops. In the process, some

cultivators became Rajputs while some

remained as they were.

Statement 1 is correct: Brahmanas and Vaisyas

who assumed the role of clan leaders or rulers

came to be known as Rajputs in the medieval

period.

Statement 2 is incorrect: Some of the chiefs of

indigenous tribes eventually became Rajputs

Note: Myth that they arose from sage Vasistha‘s

sacrificial fire at Mount Abu may have been

propagated to grant legitimacy to the right of

Rajputs to rule.

86. Ans. : (b)

Explanation:

Statement 1 is incorrect:

● The telescope was launched into space on a

Russian-built Proton-M rocket from the

Baikonur Cosmodrome in Kazakhstan.

● The four-year mission will survey the entire

sky eight times and track the evolution of the

universe and dark energy.

● The SRG will not detect gamma radiation.

Statement 2 is correct:

● It will create a three-dimensional (3D) X-ray

map of the universe.

● The telescope is the first to be sensitive to

high-energy ‗hard‘ X-rays.

● "X-rays give us a unique view of the Universe,

hidden in visible light", explains Kirpal

Nandra, director of High Energy astrophysics.

Using X-rays scientists are trying to unveil the

true beauty of the hidden Universe.

Statement 3 is correct:

● The Spectrum-Roentgen-Gamma telescope

aims to detect up to 3 mn supermassive black

holes — many of which will be new to science.

● It will also detect X-rays from 700,000 stars in

the Milky Way.

87. Ans. : (a)

Explanation:

Statement 1 is correct : Adur Razzaq, an

ambassador sent by the ruler of Persia to Calicut

(present-day Kozhikode) in the fifteenth century,

was greatly impressed by the fortifications, and

mentioned seven lines of forts. These encircled

not only the city but also its agricultural

hinterland and forests. The outermost wall

linked the hills surrounding the city. The

massive masonry construction was slightly

tapered. What was most significant about this

fortification is that it enclosed agricultural tracts.

Abdur Razzaq noted that ―between the first,

second and the third walls there are cultivated

fields, gardens and houses‖.

Statement 2 is incorrect : No mortar or

cementing agent was employed anywhere in the

construction. The stone blocks were wedge

shaped, which held them in place, and the inner

portion of the walls was of earth packed with

rubble. Square or rectangular bastions projected

outwards.

Statement 3 is incorrect :

● One of the most beautiful buildings in the

royal centre is the Lotus Mahal, so named by

British travellers in the nineteenth century.

While the name is certainly romantic,

historians are not quite sure what the building

was used for. One suggestion, found in a map

drawn by Mackenzie, is that it may have been

a council chamber, a place where the king met

his advisers.

● The mahanavami dibba is located on one of

the highest points in the city, the

―mahanavami dibba‖ is a massive platform

rising from a base of about 11,000 sq. ft to a

height of 40 ft. Rituals associated with the

structure probably coincided with

Mahanavami. The Vijayanagara kings

displayed their prestige, power and suzerainty

on this occasion. The ceremonies performed on

the occasion included worship of the image,

worship of the state horse, and the sacrifice of

buffaloes and other animals. Dances, wrestling

matches, and processions of caparisoned

horses, elephants and chariots and soldiers,

as well as ritual presentations before the king

and his guests by the chief nayakas and

subordinate kings marked the occasion.

Page 23: ALL INDIA PRELIMS TEST SERIES - 2020 · music of this form is a simple melodic extension of the raga in which it is composed. The Varnam is a beautiful creation of musical craftsmanship

22 AIPTS 2020 (HIS - 08) (E) Answer Key Byju’s Classes: 9873643487

88. Ans. : (a)

Explanation:

Statement 1 is correct: Popular movements

based on Tantra in north India and Bhakthi in

south India disregarded caste inequalities. Tantra

is an accumulation of practices and ideas

characterised by centrality of ritual and recitation

of mantra. It accords high value to the physical

body and reevaluates sensual experiences in a

positive light. Bhakti attaches great value to

devotion and love for a personal god or a

representational god by a devotee.

Statement 2 is incorrect: Sankara‘s philosophy

propounded the path of knowledge which was

difficult for the masses to comprehend. It was

Ramanuja‘s philosophy that bridged the gap

between the Vedas and the Bhakthi philosophy.

His philosophy asserts that god and soul are

different and that devotion is the way for souls to

achieve the state of liberation. For Sankara god

and soul are one and that knowledge is what

brings liberation.

89. Ans. : (a)

Explanation :

Statement 1 is correct : The average price

prevailing over the last 10 years was used while

calculating the state‘s share. 1/3rd of the average

produce was the state‘s share but the state

demand was stated in cash.

Statement 2 is incorrect : Rather than the

present prices, the price prevailing over the period

of 10 years was taken. If the price prevalent in a

particular year would have been used, the state

would lose out in case of inflation and in case of

deflation, the demand from peasants would be too

high.

Additional Information : Later, the state's share

was calculated based on local productivity and

local prices, thereby making the system even

better.

90. Ans. : (c)

Explanation:

● Developed by Naval Science and Technological

Laboratory (NSTL), a premier laboratory of

DRDO, the indigenously-built heavyweight

anti-submarine torpedo Varunastra has been

successfully inducted in the navy, making

India one of the eight countries to have the

capability to design and build such a system.

● It is an anti-submarine electric torpedo

equipped with automatic and remote-

controlled guidance systems. The weapon

system uses its own intelligence in tracing the

target.

● The weapon has been jointly developed by the

Naval Science and Technology Laboratory

(NTSL), Visakhapatnam and the Bharat

Dynamics Limited -BDL (Hyderabad).

● Having almost 95 per cent indigenous content,

Varunastra, costing about Rs 10-12 crore per

unit, is capable of targeting quiet and stealthy

submarines, both in deep and littoral waters

in an intense counter-measure environment.

91. Ans. : (a)

Explanation :

Statement 1 is correct : Generally, the lowest

rank was 10 and the highest rank 5000. Though

we have an instance of rank of 7000 being

provided. This was granted depending upon the

importance that was granted to some individual by

the emperor himself. Important nobles were given

a high rank.

Statement 2 is correct : Each zat had 3

categories

In category 1, sawar= zat

In category 2, sawar>= ½ zat

In category 3, sawar<½ zat

This ensured that there were enough cavalrymen

commanded by the mansabdars at the time of the

emperor‘s requirement.

Statement 3 is incorrect : Every mansabdar was

expected to maintain two horses per cavalryman

as horses had to be rested while on the march and

replacements were necessary in times of war. This

was however changed slightly during the reign of

Shah Jahan in order to deal with the resource

crunch.

Additional Information : The contingents of the

Mughal army were mixed ones. Each contingent of

nobles were to be drawn from Mughals,

Hindustanis (Indian Muslims), Rajputs and

Pathans. This weakened tribalism and

parochialism.

92. Ans. : (b)

Explanation:

Statement 1 is incorrect: When Alauddin Khalji

attacked Gujarat, he had not spared the Muslim

merchants of Cambay. There were no religious

considerations involved.

Statement 2 is correct: Control over territories in

Gujarat would have given the Turkish rulers

access to seaports and trade routes connecting the

west coast and the Ganga valley

Statement 3 is correct Horses were brought from

west Asia to the sea ports of Gujarat. Since cavalry

was very important for medieval warfare,

availability of horses was crucial for the

sustenance of Turkish rule. Control over western

ports would have ensured their availability.

Page 24: ALL INDIA PRELIMS TEST SERIES - 2020 · music of this form is a simple melodic extension of the raga in which it is composed. The Varnam is a beautiful creation of musical craftsmanship

23 AIPTS 2020 (HIS - 08) (E) Answer Key Byju’s Classes: 9873643487

Additional Information : Many temples and

buildings including the rebuilt Somnath temple

were sacked during the attack of Gujarat by

Alauddin Khalji. Destruction of temples were

meant to assert the political authority of rulers,

not to supplant the religion of the natives.

93. Ans. : (a)

Explanation:

Statement 1 is correct: Turkish rule had

extended upto Kampili in Karnataka when

Muhammad bin Tughlaq ascended the throne of

Delhi Sultanate.

Statement 2 is incorrect: Though Malwa (parts of

MP and Rajasthan) was under Turkish rule its

capital Mandu was not preferred by Muhammad

bin Tughlaq as his capital. Devgiri, where he was

stationed as a prince, was opted by him as the

centre of Turkish rule for better access to the

southern territories.

Additional Information : The expansion of Delhi

Sultanate created administrative and financial

problems to the Sultans. The creation of a second

capital did not prove successful but it led to

cultural assimilation of Deccan with north India.

94. Ans. : (a)

Explanation:

Statement 1 is incorrect.

The Bill regulates the use of DNA technology

for establishing the identity of persons in

respect of matters listed in a Schedule. These

include,

1. Criminal matters (offences under the IPC,

1860)

2. Civil matters (parentage disputes,

transplantation of human organs etc).

Statement 2 is correct.

Written consent is required from individuals to

collect DNA samples from them.

However consent is not required for offences

with punishment of more than 7 years of

imprisonment or death.

Not all criminal offences have more than 7

years of imprisonment. For example death

caused by negligence (Section 304A) has a

maximum penalty of seven years or less.

95. Ans. : (c)

Explanation: Fatehpur Sikri was a town planned

as an administrative unit consisting of public

buildings as well as private residence in close

proximity. This is a cultural UNESCO World

Heritage Site in India. It also served as the capital

of the Mughal Empire for a short period of time

(from 1572 to 1585).

Statement 1 is correct :

The city of Fatehpur Sikri was founded as a

token of gratitude to Sheikh Salim Chist

Akbar's spiritual preceptor. He foretold that

Akbar would have three sons who would

survive after the sad demise of many children

in infancy.

It is a square chamber, having a screened

verandah, containing latticed panels of

exquisite design, and is an elegant structure.

Statement 2 is correct :

Only one building at Fatehpur Sikri is built of

white marble, the tomb of Sheikh Salim Chisti,

Akbar's spiritual preceptor.

Shaikh Salim Chishti (1478-1572) is one of the

most revered Sufi saints of the Mughal period

in India.

96. Ans. : (c)

Explanation:

The Mahishasura Mardini cave temple in

Mahabalipuram takes its name from the huge

sculpture panel on its wall celebrating the feat

of the goddess Durga, a form of the mother

goddess Shakti.

This cave depicts a great masterpiece showing

the great goddess Durga engaged in a fierce

battle with the buffalo headed demon aided by

their respective armies. Riding on her lion she

is rushing at the powerful demon with great

courage. He is moving away, yet watching for a

moment to attack.

It represents the eternal struggle between the

forces of good and evil, in which the good

ultimately triumphs. The dramatic movement,

emotional intensity and visual realism noticed

in this sculpture are worthy of a master

craftsman.

97. Ans. : (b)

Explanation:

Nagara style of Architecture: It is the type of

temple architecture that developed north of

the Vindhyas. It evolved during the Gupta

period. The nagara style is characterised by a

curved spiral roof over the deity room. Many

temples built during the medieval period

followed this style.

Kandariya Mahadeo Temple, Khajuraho, Sun

Temple, Konark, Somnath Temple, Saurashtra

were built by Chandella, Eastern Ganga and

Solanki rulers respectively. They were Rajput

clans. The nagara style was followed by the

Rajputs in north india and the Deccan The

Kailashnath temple was built by a

Rashtrakuta ruler. It has traces of Pallava and

Chalukyan styles of architecture

Page 25: ALL INDIA PRELIMS TEST SERIES - 2020 · music of this form is a simple melodic extension of the raga in which it is composed. The Varnam is a beautiful creation of musical craftsmanship

24 AIPTS 2020 (HIS - 08) (E) Answer Key Byju’s Classes: 9873643487

Kandariya Mahadeo Temple, Khajuraho : It is

the largest and most ornate Hindu temple in the

medieval temple group found at Khajuraho in

Madhya Pradesh, India. It is considered one of the

best examples of temples preserved from the

medieval period in India. The Kandariya Mahadeva

temple was built during the reign of Chandella

king, Vidyadhara in the 11th century.

Sun Temple, Konark : It was built in the 13th-

century.The temple is attributed to king

Narasingha Deva I of the Eastern Ganga Dynasty.

Dedicated to the Hindu Sun God Surya, what

remains of the temple complex has the appearance

of a 30 m high chariot with immense wheels and

horses, all carved from stone.

Somnath Temple, Saurashtra : It was built by

Solanki Rajputs( the Chalukya dynasty of Gujarat)

in the 11th century.The temple is dedicated to

Lord Shiva.

The Kailashnath temple : A megalith carved from

a rock cliff face, it is considered one of the most

remarkable cave temples in the world because of

its size, architecture and sculptural treatment.

Most of the excavation of the temple is generally

attributed to the eighth century Rashtrakuta king

Krishna I.

98. Ans. : (c)

Explanation:

Airports Authority of India (AAI) is working

towards the implementation of a Centrally

controlled Air Traffic Flow Management

(CAFTM) system across the country's airspace

and major airports, particularly those with

high traffic density, in its bid to cut travel time

for fliers.

The system is primarily meant to address the

balancing of capacity against the demand to

achieve optimum utilization of the resources.

The new system would integrate data from

airlines, airports and air-traffic agencies and

inform airlines about the time planes will be

parked at the destination airport well before

departure.

99. Ans. : (a)

Explanation :

Statement 1 is correct : The jagir system was

followed to remunerate the mansabdars. The land

revenue of a jagir was assigned to a mansabdar.

However, sometimes the mansabdars were also

paid in cash.

Statement 2 is incorrect : A jagir did not confer

any hereditary rights on the holder, or disturb any

of the existing rights in the area. It only meant

that the land revenue which was due to the state

was to be paid to the jagirdar

Additional Information : This is an old system

which was also used during the Delhi Sultanate

with the lands being designated known as iqtas

and the holders known as iqtadars.

100. Ans. : (c)

Explanation:

Statement 1 is correct: The Treaty of Purandar

(1665) was signed between Raja Jai Singh of

Amber and Shivaji. After the loss of Surat, the

chief port of the Mughals, from the attack of

Shivaji, Aurangazeb sent Raja Jai Singh of Amber

to fight against Shivaji. Raja Jai Singh of Amber

succeeded in besieging the Purander fort where

Shivaji lodged his family and treasure. Shivaji

opened negotiations with Jai Singh and the Treaty

of Purandar was signed in 1665.

Statement 2 is correct: Under the Treaty of

Purandar (1665), Mughals recognized the right of

Shivaji to hold certain parts of the Bijapur

kingdom.

Statement 3 is incorrect: According to the treaty,

Shivaji had to surrender 23 forts to the Mughals

out of 35 forts held by him. The remaining 12 forts

were to be left to Shivaji on condition of service

and loyalty to Mughal empire.